Đến nội dung

song_ha nội dung

Có 279 mục bởi song_ha (Tìm giới hạn từ 29-04-2020)



Sắp theo                Sắp xếp  

#8007 giới hạn về e

Đã gửi bởi song_ha on 14-02-2005 - 10:54 trong Dãy số - Giới hạn

biện lụân sự hội tụ của dăy theo



#8034 pt nghiem nguyen

Đã gửi bởi song_ha on 14-02-2005 - 15:22 trong Số học

cũng có bài dễ hôn (mốc_đen) :wacko:
mong được trao đổi về lớp bài tổng quát



#8036 Bài hay

Đã gửi bởi song_ha on 14-02-2005 - 15:35 trong Phương trình - Hệ phương trình - Bất phương trình

tìmthỏa
dễ và hơi bị quen



#9047 Hàm trên tập các số nguyên dương

Đã gửi bởi song_ha on 21-02-2005 - 23:24 trong Phương trình - Hệ phương trình - Bất phương trình

Gửi quan vũ:
xét f(3k+r)=a(k) với r=0,1,2
can CM a(k):cấp số nhân thayvào có f(n)=1



#9048 Tim Han So Sau

Đã gửi bởi song_ha on 21-02-2005 - 23:38 trong Phương trình - Hệ phương trình - Bất phương trình

xét g(x)= f(x)-8x.x/7
có g(x)=g(x/2)/2 đặt h(x)=xg(x) với x khác 0 sẽ có
h(x) = h(x/2) qua fép đặt hàm mũ quy về hàm tuần hàm



#9056 bán kính

Đã gửi bởi song_ha on 22-02-2005 - 00:21 trong Công thức lượng giác, hàm số lượng giác

nhận diện tam giác biết là
2cotg(B/2)+cotg(C/2)=4sinA+căn(3)



#9414 BDT khó

Đã gửi bởi song_ha on 23-02-2005 - 22:20 trong Bất đẳng thức - Cực trị

ks hàm có gtnn=(1/e)^(1/e)



#9420 bán kính

Đã gửi bởi song_ha on 23-02-2005 - 22:30 trong Công thức lượng giác, hàm số lượng giác

Viết lại giúp bạn cho dễ nhìn ngheng.

Nhận tam giác biết là :


xin lỗi cô "péc" 'nghen' tại anh dốt tin nhưng ko biết tex chắc
em xơi ngon bài đó nhưng anh muốn có 1lời giải thật đơn giản cho nó
nó nằm trong 1cđ BĐT của anh.Thường thôi!



#9427 cos va tg

Đã gửi bởi song_ha on 23-02-2005 - 22:41 trong Công thức lượng giác, hàm số lượng giác

đánh giá
nhân vào đưa về dùng js là xong



#9433 bổ đề qtr cho bài toán hay

Đã gửi bởi song_ha on 23-02-2005 - 22:52 trong Số học

Mình có 1bài hay dẫn đến ptr kiểu Pell mong các bạn giải quyết
ptr là
mình ngờ là nó vô No nếu ai giải quyết được thì xin hậu tạ!

DDTH



#9451 BDT.

Đã gửi bởi song_ha on 23-02-2005 - 23:30 trong Bất đẳng thức - Cực trị

sợ cô péc nhầm đề chứ thế thì nên thay 2 bởi 1 cho khoẻ



#9464 bài ko' tầm thường' bị xào lại

Đã gửi bởi song_ha on 24-02-2005 - 00:32 trong Số học

cho ham http://dientuvietnam.net/cgi-bin/mimetex.cgi?f:N-->N biết
http://dientuvietnam.net/cgi-bin/mimetex.cgi?f(n) ko chia hết cho http://dientuvietnam...imetex.cgi?2003



#9553 cos va tg

Đã gửi bởi song_ha on 24-02-2005 - 19:37 trong Công thức lượng giác, hàm số lượng giác

cho em hỏi nếu dùng JS thì cần điều kiện hàm lồi lõm,còn hàm sin A thì lồi lõm ra sao mà có thể dùng JS được



Ô kìa dùng JS với với chứ!



#9555 Tim ham f .

Đã gửi bởi song_ha on 24-02-2005 - 20:40 trong Phương trình - Hệ phương trình - Bất phương trình

Tim tat ca cac ham so f : R -> R thoa 3 tinh chat sau :

1 / f(0) =0    , f (1) =  1

2/  :sum  a , b  :sum  R  , a < b =>  f(a)    :geq  f(b)

  3/ f(a) + f(b) = f(a) .f(b) + f(a+ b - ab )  :sum  a , b  :in  R a < 1 < b.



Đặt http://dientuvietnam...mimetex.cgi?h(x)=1-f(x+1) tức là http://dientuvietnam...mimetex.cgi?f(x)=1-h(x-1)
sẽ có http://dientuvietnam...mimetex.cgi?h(x).h(y)=h(xy) với
xong em tự đi tiếp nhé( T/B:các bài của em khá hay!)



#9636 BDT.

Đã gửi bởi song_ha on 25-02-2005 - 14:42 trong Bất đẳng thức - Cực trị

Cho a , b , c là các số dương thỏa mãn :


Chứng minh BDT sau:


Vì ab+bc+ca-abc<ab+bc+ca <= a^2+b^2+c^2 < a^2+b^2+c^2+abc=1
đúng em chép đề nhầm ko?
anh đoán đề đúng ra là thay 1bởi 4 trong đẳng thưc ở gt
CÁCH GIẢI:
1/ dùng bđt "mũi hếch'' & "xúc"
2/ đặt a=2cosA, b=2cosB, c=2cosC với tam giác ABC nhọn
3/ Đưa về bài VMO_96 .ở vmo gthiết ab+bc+ca+abc có
thể xào thành x^2+y^2+z^2+xyz=4 nhờ đặt ab=x^2...
đây là bài anh bị 1thầy ở CTB thử hồi anh còn thất ngiệp về đấy kiếm ăn
Nếu để đẳng thức cũ ở gt
bỏ vế>=0 đi với a,b,c thực liệu đúng ko cô péc



#9638 Thơ của người học Toán

Đã gửi bởi song_ha on 25-02-2005 - 14:51 trong Quán văn

"từ cấm""từ cấm""từ cấm""từ cấm""từ cấm""từ cấm""từ cấm"



#10156 BDT.

Đã gửi bởi song_ha on 28-02-2005 - 13:46 trong Bất đẳng thức - Cực trị

Hôm nay em hỏi lại đề đúng là như vậy ( thay 1 bởi 4).Anh tinh ý quá . Còn BDT "mũi hếch " và "xúc" là như nào vậy anh.
Cảm ơn anh.


à đấy là BĐT murihead,và shurr ấy mà.Em có thể xem nội dung trong
quyển BĐT của Hardy& Polia.vì anh ko biết tex nên ko post luôn cho em được
Nếu hôm nào rỗi anh bảo mấy đứa học trò anh post cho (bọn nó biết tex)nhưng chắc em biết nó
rồi chứ



#10161 số học

Đã gửi bởi song_ha on 28-02-2005 - 14:22 trong Số học

y<=1nhờ bạn tự xét
với y>=2 có x chia hết cho3.Đặt x+1980=3.X pt đưa về
(X+4)^2=3^(y-2)+7 vì bìnhphương chỉ chia 4 dư 0và 1nên
y:chẵn ta có 7=(a-b)(a+b)lý luận ước có kết quả



#10162 bài rất khó

Đã gửi bởi song_ha on 28-02-2005 - 14:29 trong Số học

đây là bài toán mà anh bạntôi(đinhvKhâm)đố tôi giảiđược
gọi S(n) là tổng các chữ số của n
tìm a sao cho
xin chia sẻ cùng các bạn
T/B:trên mathlinks có 1 bài tương tự

----

Bạn có bổ xung điều kiện gì cho a không? Nguyên, Nguyên Dương hay Có thể Thực?



#10163 1 đánh giá lượng giác

Đã gửi bởi song_ha on 28-02-2005 - 14:43 trong Bất đẳng thức - Cực trị

đây là bài toán do bạnLê fi hùng gv ch hàtĩnh nhờ tôi giải hộ với
nhắn nhủ nó đã tồn tại ở đó 1 năm.giải xong tôi chỉ biết trách mình
đã quá đỗi ngây ngô.xin đưa ra góp vui cùng các bạn(nguyên văn đề)

CMR:với mọi tam giác ABCcó
cosA+cos(B/2)+cos(C/3)<=3căn(3)/2



#10167 bài rất khó

Đã gửi bởi song_ha on 28-02-2005 - 14:55 trong Số học

đây là bài toán mà anh bạntôi(đinhvKhâm)đố tôi giảiđược
gọi S(n) là tổng các chữ số của n
tìm a sao cho
xin chia sẻ cùng các bạn
T/B:trên mathlinks có 1 bài tương tự

----

Bạn có bổ xung điều kiện gì cho a không? Nguyên, Nguyên Dương hay Có thể Thực?

xin lỗi nhé a:nguyên dương



#10170 1 đánh giá lượng giác

Đã gửi bởi song_ha on 28-02-2005 - 15:11 trong Bất đẳng thức - Cực trị

đây là bài toán do bạnLê fi hùng gv ch hàtĩnh nhờ tôi giải hộ với
nhắn nhủ nó đã tồn tại ở đó 1 năm.giải xong tôi chỉ biết trách mình
đã quá đỗi ngây ngô.xin đưa ra góp vui cùng các bạn(nguyên văn đề)

       CMR:với mọi tam giác ABCcó
cosA+cos(B/2)+cos(C/3)<=3căn(3)/2

Chỉ hints thôi nhé:

1. Với tam giác tù bất đẳng thức trên đúng tự minh
2. Với 2 số . CMR
trong đó
3. Chứng minh với ta có

4. Chứng minh rằng với ta có


Khi nào bạn đọc về hàm lồi, bạn sẽ nhìn mấy bất đẳng thức loại này "tầm thường" hơn


vũ hung giải còn dài đấy ko cần ngiêm trọng vậy đâu BĐT đó
có đúng đâu mà giải chỉ cần xét gtlnVT>=gtln(cos(pi/2-x)+cosx/2+cos(pi/6))
gtln kia>3cẳn3/2 bằng việc thay x=pi/3 vào đạo hàm thôi mà
mình cũng biết đôi chút về hàm lồi từ hồi học HV vuhung à.



#10371 bài ko' tầm thường' bị xào lại

Đã gửi bởi song_ha on 01-03-2005 - 19:28 trong Số học

sao ko thấy ai hồi âm bài này của tôi nhỉ lẽ nào nó quá đơn giản
với mọi người
thế thì xin thay bởi nguyên tác vậy
cho <img src="http://dientuvietnam...in/mimetex.cgi? x_{n+1} = x_{n} ^{4} -4. x_{n} ^{2} +2; x_{1}=14 " $

cmr: <img src="http://dientuvietnam...in/mimetex.cgi? x_{n}" $ ko chia het cho 2003 voi moi n



#10381 Bài hay

Đã gửi bởi song_ha on 01-03-2005 - 20:41 trong Phương trình - Hệ phương trình - Bất phương trình

tậpN chứa cả 0 cơ mà sao mọi người thấy dễ quá hay sao vậy
để tớ ế hàng lâu quá ko lẽ lại tự thanh lý
nếu ai giải được xin giúp đỡ bài khác vẫn với đẳng thức hàm đó
nhưng thay N bởi N*



#10395 số học

Đã gửi bởi song_ha on 01-03-2005 - 21:46 trong Số học

Gửi MR'MATH
với y<0 em tự giải
với y>0
đưa pt về (2x+3)^2 - 4.3^y = 4003.3967
_ nếu y:chẵn đưa về pt tổng bph rồi luận ước(tự giải giúp anh nhé)

_Nếu y:lẻ sd định lý FERMAt nhỏ sẽ có 3^2001-1chia hết cho 4003:số ngtố
mà nếu sd thặng dư bậc 2 thì3^((p-1)/2) - 1 chia hết cho p:số ngtố khi chỉ khi p=12k+1hoặc p=12k-1
trong khi 4003=12k+7 vậy vô no ylẻ
(t/b:em chế tạo hay quá sinh năm 81 à)
à quên tiện vì 2003 & 4003 là số nguyên tố em thử chứng tỏ cho anh em
diễn đàn 1 lời giải sơ cấp cho đl DỈICKLE ''nhái'' này xem
" trong cấp số cộng 2000k+3 có vô số số nguyên tố"
đố cho vui thôi vì thấy em siêu số đừng nghĩ gì ko đẹp nhé